5

I'm reading a book where quantum interference is explained with a common experiment, where photons are thrown to some apparatus formed by two beam splitters:

enter image description here

Describing the problem using linear algebra, we could assign some vectors for the two possible paths that the photon could take after the first beam splitter, for example

$$ \text{Red path} \equiv \left(\begin{array}{c}1\\0\end{array}\right) \quad \text{Green path} \equiv \left(\begin{array}{c}0\\1\end{array}\right) $$

Then, if we say that the photon starts in the state of the red path, the quantum state after the first beam splitter is

$$ \lvert\psi\rangle = \dfrac{1}{\sqrt{2}} \left(\begin{array}{cc}1&i\\i&1\end{array}\right)\left(\begin{array}{c}1\\0\end{array}\right) = \dfrac{1}{\sqrt{2}}\left(\begin{array}{c}1\\i\end{array}\right) = \dfrac{1}{\sqrt{2}}\left(\begin{array}{c}1\\0\end{array}\right) + \dfrac{i}{\sqrt{2}}\left(\begin{array}{c}0\\1\end{array}\right) $$

Mathematically, this is nice because it gives 50% of probabilities for the photon to be found in each path since

$$ \left|\dfrac{1}{\sqrt{2}}\right|^2 = \dfrac{1}{2} \quad \text{and} \quad \left|\dfrac{i}{\sqrt{2}}\right|^2 = \dfrac{1}{2} $$

However, I don't see why the book represents the beam splitter using the imaginary number $i$. Wouldn't it be the same physically speaking (in terms of the resultant probability) if we represent the beam splitter with the usual Hadamard's gate

$$ \dfrac{1}{\sqrt{2}} \left(\begin{array}{cc}1&1\\1&-1\end{array}\right) $$

Is there any reason (historical, physical, ...) that could make the author choose the matrix with the $i$ components?

SrJaimito
  • 591
  • Check out the first answer here, I think it should help: https://physics.stackexchange.com/questions/193169/beam-splitter-in-q-m-and-c-m-formalism – Miriam Mar 24 '19 at 09:15

0 Answers0